e est un nombre irrationnel

Voici une preuve assez simple de l’irrationalité du nombre \(e\) (considérant une connaissance préalable des factorielles et des séries géométriques). À l’instar de certaines démonstrations parfois faites avec les élèves montrant que \(\sqrt{2}\) et \(\log_{10}(2)\) sont des nombres irrationnels, il s’agit d’une preuve par l’absurde.

En se rappelant que le nombre \(e\) est égal à la série infinie suivante \[e = \frac{1}{0!}+\frac{1}{1!}+\frac{1}{2!} + \frac{1}{3!} + \ \dots \ + \frac{1}{n!} + \ \dots\]avec \(0! = 1\), on appelle \(S_{n}\) la somme partielle des termes de cette série.  C’est-à-dire la somme des \(n\) premiers termes de cette série telle que \[S_{n} = \frac{1}{0!}+\frac{1}{1!}+\frac{1}{2!}+\frac{1}{3!}+ \ \dots \ +\frac{1}{(n-1)!}+\frac{1}{n!}\]Remarquons que pour tout \(n = 1, \ 2, \ 3, \ 4, \ \dots \ \) on a \[S_{n}<e\]Cela implique aussi que \[0< e-S_{n}\]Que reste-t-il lorsqu’on soustrait \(S_{n}\) à \(e\) ?  Il reste \[0<e-S_{n}=\frac{1}{(n+1)!}+\frac{1}{(n+2)!}+\frac{1}{(n+3)!}+\ \dots\]ce qui est égal à \[0<e-S_{n}=\frac{1}{(n+1)!}+\frac{1}{(n+1)!(n+2)} + \frac{1}{(n+1)!(n+2)(n+3)}+ \ \dots\]En effectuant une mise en évidence simple, on obtient \[0<e-S_{n}=\frac{1}{(n+1)!}\left(1+\frac{1}{n+2}+\frac{1}{(n+2)(n+3)}+ \ \dots \right)\]Nous posons alors l’inéquation suivante, étape clé de la démarche,\begin{align*}0<e-S_{n}&=\frac{1}{(n+1)!}\left(1+\frac{1}{n+2}+\frac{1}{(n+2)(n+3)}+ \ \dots\right) \\ \\ &<\frac{1}{(n+1)!}\left(1+\frac{1}{n+1} + \frac{1}{(n+1)(n+1)} + \ \dots\right)\end{align*}puisque, toujours avec \(n = 1,\ 2, \ 3, \ 4, \ \dots \ \) les dénominateurs de chaque terme étant respectivement strictement plus petits, les fractions sont respectivement strictement plus grandes \begin{align*}\frac{1}{n+1}&>\frac{1}{n+2} \\ \\ \frac{1}{(n+1)(n+1)}&>\frac{1}{(n+2)(n+3)} \\ \\ \frac{1}{(n+1)(n+1)(n+1)} &> \frac{1}{(n+2)(n+3)(n+4)}\\ \\ &\dots \end{align*}Or, cette nouvelle expression est constituée d’une série géométrique infinie (dans la parenthèse).  Le premier terme \(a\) de cette série géométrique est \(1\) et la raison \(r\) de cette série géométrique est \[r=\frac{1}{n+1}\]Or, comme \(n = 1,\ 2, \ 3, \ 4, \ \dots \ \) il se trouve très certainement que \[r<1\]et on peut donc exprimer la somme de cette série géométrique infinie de façon succincte, c’est \[\frac{a}{1-r}\]On trouve \begin{align*}0<e-S_{n}&=\frac{1}{(n+1)!}\left(1+\frac{1}{n+2}+\frac{1}{(n+2)(n+3)}+\ \dots\right)\\ \\ &<\frac{1}{(n+1)!}\left(1+\frac{1}{n+1}+\frac{1}{(n+1)(n+1)}+\ \dots \ \right) = \frac{1}{(n+1)!}\left(\frac{1}{1-\frac{1}{n+1}}\right)\end{align*}En reprenant seulement ce qui nous intéresse \[0<e-S_{n}<\frac{1}{(n+1)!}\left(\frac{1}{1-\frac{1}{n+1}}\right)\]et en mettant sur dénominateur commun \[0<e-S_{n}<\frac{1}{(n+1)!}\left(\frac{1}{\frac{n+1}{n+1}-\frac{1}{n+1}}\right)\]et en effectuant la soustraction, on obtient \[0<e-S_{n}<\frac{1}{(n+1)!}\left(\frac{1}{\frac{n}{n+1}}\right)\]et enfin en simplifiant \[0<e-S_{n}<\frac{1}{(n+1)!}\left(\frac{n+1}{n}\right)\]Comme \[(n+1)! = n!\,(n+1)\]le numérateur \(n+1\) s’annule avec le même facteur du dénominateur.  Il reste \[0<e-S_{n}<\frac{1}{n!\cdot n}\]En multipliant par \(n!\) de chaque côté (\(n!\) étant strictement positif, on ne change pas les signes \(<\) de côté) on obtient \[0<n!\,(e-S_{n})<\frac{1}{n}\]Notons à ce moment que cette inégalité tient pour tout \(n = 1, \ 2, \ 3, \ 4, \ \dots \ \)

Supposons que \(e\) soit un nombre rationnel.  Il pourrait alors s’écrire comme une fraction d’entiers.  Sachant que la valeur de \(e\) est entre \(2\) et \(3\), supposons que la fraction réduite représentant le nombre \(e\) soit \[e = \frac{p}{q}\]avec \[p, \, q \in \mathbb{N}\]et \[\text{pgcd}(p,\, q) =1 \]Cette supposition nous mènera à une contradiction.

Puisque l’inégalité ci-haut tient pour tout \(n = 1, \ 2, \ 3, \ 4, \ \dots \ \) choisissons \(n\) tel que \[n>q\]Certainement,\[q>1\]sans quoi \(e\) serait un entier (ce qui n’est clairement pas le cas).  On en déduit donc que \[n>1\]et que \[\frac{1}{n}<1\]c’est-à-dire une fraction strictement plus petite que \(1\).  Dans l’inégalité suivante \[0<n!\,(e-S_{n})<\frac{1}{n}<1\]l’expression du milieu \[n!\,(e-S_{n})\]est comprise en \(0\) et \(1\) (exclusivement) et ne peut donc pas être un nombre entier.  En remplaçant \(e\) par la fraction réduite qui lui est égale, obtient \[n!\,(e-S_{n}) = n!\left(\frac{p}{q}-S_{n}\right)\]puis en développant \(S_{n}\) \begin{align*}n!\,(e-S_{n})&= n!\left(\frac{p}{q}-S_{n}\right) \\ \\ &=n!\left(\frac{p}{q}-\left(\frac{1}{0!}+\frac{1}{1!}+\frac{1}{2!}+ \ \dots \ +\frac{1}{n!}\right)\right) \\ \\ &=n!\left(\frac{p}{q}\right)-n!\left(1+1+\frac{1}{2!}+\ \dots \ + \frac{1}{n!}\right)\end{align*}Enfin réexprimant le premier terme et en distribuant \(n!\) dans la deuxième parenthèse on obtient \[n!\, (e-S_{n}) = \left(\frac{n!}{q}\right)p-\left(n! + n! + \frac{n!}{2!}+\ \dots \ + \frac{n!}{n!}\right)\]Or, on a choisit \[n>q\]Cela implique que \(q\) divise \(n!\) puisque \[n! = 1 \cdot 2 \cdot 3 \cdot \ \dots \ \cdot q \cdot \ \dots \ \cdot n\]Le premier terme \[\left(\frac{n!}{q}\right)p\]est donc un nombre entier.  Par ailleurs, le même argument s’applique à chaque terme à l’intérieur de la parenthèse : \(n!\) est certainement un entier, \(2!\) divise \(n!\), c’est donc aussi un entier,  \(3!\) divise \(n!\), encore là un entier…  Et ce jusqu’au dernier terme, \(n!\) divisant certainement \(n!\), il s’agit encore d’un entier.  L’expression entre parenthèse \[\left(n! + n! + \frac{n!}{2!} + \ \dots \ + \frac{n!}{n!}\right)\]est donc aussi un nombre entier.  Aussi surprenant que cela puisse paraître, l’expression \[\left(\frac{n!}{q}\right)p-\left(n!+n!+\frac{n!}{2!}+ \ \dots \ + \frac{n!}{n!}\right)\]est un entier.  Or comme \[n!\, (e-S_{n})\]n’est pas un entier, et que nous avions plus haut \[n! \, (e-S_{n}) = \left(\frac{n!}{q}\right)p-\left(n!+n!+\frac{n!}{2!}+ \ \dots \ + \frac{n!}{n!}\right)\]nous obtenons la contradiction recherchée !  Le nombre \(e\) est irrationnel et ne peut s’exprimer comme un quotient d’entiers.

5 thoughts on “e est un nombre irrationnel

  1. Une première questio qui se pose: l’écriture de e à l’aide des séries exige du lecteur des connaissances de haut niveau en maths. Par conséquent la preuve par absurde de l’irrationnalité de e ne lui poserait pas de problème. Elle est plus onvaincante que celle avec les séries. Cette dernière est formelle et éclairante mais pas convaincante.
    De plus dans votre preuve il y a aussi raisonnement par absurde

  2. Bonjour Cherif Zhor,

    vous avez un bon point. Si on veut montrer que (1 + 1/k)^k lorsque k tend vers l’infini est égal à la somme des 1/(n!) lorsque n tend vers l’infini, alors oui, cela exige du lecteur des connaissances mathématiques de plus haut niveau (donc effectivement, aucun problème à comprendre la preuve).

    Cependant, si on ne considère que la série des 1/(n!), il n’est pas difficile de montrer que cette série converge vers une valeur entre 2 et 3. On pourrait donc sans problème, dans un premier temps, s’attarder à cette valeur et montrer qu’elle est irrationnelle. La suite viendrait éventuellement…

    Quant au raisonnement par l’absurde, c’était bien sûr voulu ! C’était le but de la chose !

  3. Merci beaucoup, c’est bien détaillé et de fait très agréable à suivre et à lire.
    La méthode m’a permis de démontrer que la série des ((-1)^n)/(2n!) convergeait vers un irrationnel en raisonnant de façon similaire. C’est une démonstration que je ne parvenais pas à faire depuis un moment et je n’ai trouvé nulle par ailleurs de quoi m’aider.

  4. Ok j’ai suivi le raisonnement et j’ai compris donc merci.
    Pourquoi avoir écrit on choisit n > q
    Ca laisse entendre que vous allez tester n < q
    Or n tend vers l'infini d'apres la définition de e qui est une somme infinie de 1/n
    Donc n est forcément supérieur à n'importe quel entier q

  5. Bonjour Paul,

    ici, n est choisi pour « séparer » la série qui correspond à e :
    e = (1 + 1/1! + 1/2! + … + 1/n!) + (1/(n + 1)! + 1/(n + 2)! + … )
    La première parenthèse comporte un nombre fini de termes, c’est la somme partielle, Sn. La deuxième parenthèse contient un nombre infini de termes.

    Ce n’est pas n qui tend vers l’infini. Dans le billet, j’ai mis trois petits points, mais on aurait pu écrire (peut-être plus rigoureusement) quelque chose du genre

    e = lim k→∞ 1 + 1/1! + 1/2! + 1/3! + … + 1/n! + … + 1/k!

Leave a Reply

Your email address will not be published. Required fields are marked *

Time limit exceeded. Please complete the captcha once again.